Difference between revisions of "2020 AMC 8 Problems/Problem 4"

(Solution 2)
Line 33: Line 33:
  
 
==Solution==
 
==Solution==
We find the pattern <math>1, 6, 12, 18, \ldots</math>. The sum of the first four numbers in this sequence is <math>\boxed{\textbf{37}}</math>.
+
We find the pattern <math>1, 6, 12, 18, \ldots</math>. The sum of the first four numbers in this sequence is <math>\boxed{\textbf{(B) }37}</math>.
  
 
==Solution 2==
 
==Solution 2==
Line 39: Line 39:
 
~ junaidmansuri
 
~ junaidmansuri
  
 +
==Solution 3==
 +
Each band adds <math>1, 6, 2(6), 3(6), 4(6) \cdots,</math> so we have <math>1+6+2(6)+3(6)=1+6(6)=1+36=\boxed{\textbf{(B) }37}.</math>
 +
 +
[pog]
 
==See also==  
 
==See also==  
 
{{AMC8 box|year=2020|num-b=3|num-a=5}}
 
{{AMC8 box|year=2020|num-b=3|num-a=5}}
 
{{MAA Notice}}
 
{{MAA Notice}}

Revision as of 08:34, 18 November 2020

Three hexagons of increasing size are shown below. Suppose the dot pattern continues so that each successive hexagon contains one more band of dots. How many dots are in the next hexagon? [asy] size(250); real side1 = 1.5; real side2 = 4.0; real side3 = 6.5; real pos = 2.5; pair s1 = (-10,-2.19); pair s2 = (15,2.19); pen grey1 = rgb(100/256, 100/256, 100/256); pen grey2 = rgb(183/256, 183/256, 183/256); fill(circle(origin + s1, 1), grey1); for (int i = 0; i < 6; ++i) { draw(side1*dir(60*i)+s1--side1*dir(60*i-60)+s1,linewidth(1.25)); } fill(circle(origin, 1), grey1); for (int i = 0; i < 6; ++i) { fill(circle(pos*dir(60*i),1), grey2); draw(side2*dir(60*i)--side2*dir(60*i-60),linewidth(1.25)); } fill(circle(origin+s2, 1), grey1); for (int i = 0; i < 6; ++i) { fill(circle(pos*dir(60*i)+s2,1), grey2); fill(circle(2*pos*dir(60*i)+s2,1), grey1); fill(circle(sqrt(3)*pos*dir(60*i+30)+s2,1), grey1); draw(side3*dir(60*i)+s2--side3*dir(60*i-60)+s2,linewidth(1.25)); } [/asy]

Diagram by sircalcsalot

$\textbf{(A) }35 \qquad \textbf{(B) }37 \qquad \textbf{(C) }39 \qquad \textbf{(D) }43 \qquad \textbf{(E) }49$

Solution

We find the pattern $1, 6, 12, 18, \ldots$. The sum of the first four numbers in this sequence is $\boxed{\textbf{(B) }37}$.

Solution 2

The first hexagon has $1$ dot. The second hexagon has $1+6$ dots. The third hexagon $1+6+12$ dots. Following this pattern, we predict that the fourth hexagon will have $1+6+12+18=37$ dots $\implies\boxed{\textbf{(B) }37}$.
~ junaidmansuri

Solution 3

Each band adds $1, 6, 2(6), 3(6), 4(6) \cdots,$ so we have $1+6+2(6)+3(6)=1+6(6)=1+36=\boxed{\textbf{(B) }37}.$

[pog]

See also

2020 AMC 8 (ProblemsAnswer KeyResources)
Preceded by
Problem 3
Followed by
Problem 5
1 2 3 4 5 6 7 8 9 10 11 12 13 14 15 16 17 18 19 20 21 22 23 24 25
All AJHSME/AMC 8 Problems and Solutions

The problems on this page are copyrighted by the Mathematical Association of America's American Mathematics Competitions. AMC logo.png